Английская Википедия:Euclid's theorem

Материал из Онлайн справочника
Перейти к навигацииПерейти к поиску

Шаблон:Short description Шаблон:About

Euclid's theorem is a fundamental statement in number theory that asserts that there are infinitely many prime numbers. It was first proven by Euclid in his work Elements. There are several proofs of the theorem.

Euclid's proof

Euclid offered a proof published in his work Elements (Book IX, Proposition 20),[1] which is paraphrased here.[2]

Consider any finite list of prime numbers p1p2, ..., pn. It will be shown that at least one additional prime number not in this list exists. Let P be the product of all the prime numbers in the list: P = p1p2...pn. Let q = P + 1. Then q is either prime or not:

  • If q is prime, then there is at least one more prime that is not in the list, namely, q itself.
  • If q is not prime, then some prime factor p divides q. If this factor p were in our list, then it would divide P (since P is the product of every number in the list); but p also divides P + 1 = q, as just stated. If p divides P and also q, then p must also divide the difference[3] of the two numbers, which is (P + 1) − P or just 1. Since no prime number divides 1, p cannot be in the list. This means that at least one more prime number exists beyond those in the list.

This proves that for every finite list of prime numbers there is a prime number not in the list.[4] In the original work, as Euclid had no way of writing an arbitrary list of primes, he used a method that he frequently applied, that is, the method of generalizable example. Namely, he picks just three primes and using the general method outlined above, proves that he can always find an additional prime. Euclid presumably assumes that his readers are convinced that a similar proof will work, no matter how many primes are originally picked.[5]

Euclid is often erroneously reported to have proved this result by contradiction beginning with the assumption that the finite set initially considered contains all prime numbers,[6] though it is actually a proof by cases, a direct proof method. The philosopher Torkel Franzén, in a book on logic, states, "Euclid's proof that there are infinitely many primes is not an indirect proof [...] The argument is sometimes formulated as an indirect proof by replacing it with the assumption 'Suppose Шаблон:Math are all the primes'. However, since this assumption isn't even used in the proof, the reformulation is pointless."[7]

Variations

Several variations on Euclid's proof exist, including the following:

The factorial n! of a positive integer n is divisible by every integer from 2 to n, as it is the product of all of them. Hence, Шаблон:Nowrap is not divisible by any of the integers from 2 to n, inclusive (it gives a remainder of 1 when divided by each). Hence Шаблон:Nowrap is either prime or divisible by a prime larger than n. In either case, for every positive integer n, there is at least one prime bigger than n. The conclusion is that the number of primes is infinite.[8]

Euler's proof

Another proof, by the Swiss mathematician Leonhard Euler, relies on the fundamental theorem of arithmetic: that every integer has a unique prime factorization. What Euler wrote (not with this modern notation and, unlike modern standards, not restricting the arguments in sums and products to any finite sets of integers) is equivalent to the statement that we have[9]

<math>\prod_{p\in P_k} \frac{1}{1-\frac{1}{p}}=\sum_{n\in N_k}\frac{1}{n},</math>

where <math>P_k</math> denotes the set of the Шаблон:Mvar first prime numbers, and <math>N_k</math> is the set of the positive integers whose prime factors are all in <math>P_k.</math>

In order to show this, one expands each factor in the product as a geometric series, and distributes the product over the sum (this is a special case of the Euler product formula for the Riemann zeta function).

<math>

\begin{align} \prod_{p\in P_k} \frac{1}{1-\frac{1}{p}} & =\prod_{p\in P_k} \sum_{i\geq 0} \frac{1}{p^i}\\ & = \left(\sum_{i\geq 0} \frac{1}{2^i}\right) \cdot \left(\sum_{i\geq 0} \frac{1}{3^i}\right) \cdot \left(\sum_{i\geq 0} \frac{1}{5^i}\right) \cdot \left(\sum_{i\geq 0} \frac{1}{7^i}\right)\cdots \\ & =\sum_{\ell,m,n,p,\ldots \geq 0} \frac{1}{2^\ell 3^m 5^n 7^p \cdots} \\ & =\sum_{n\in N_k}\frac{1}{n}. \end{align} </math>

In the penultimate sum every product of primes appears exactly once, and so the last equality is true by the fundamental theorem of arithmetic. In his first corollary to this result Euler denotes by a symbol similar to <math>\infty</math> the « absolute infinity » and writes that the infinite sum in the statement equals the «  value » <math>\log\infty</math>, to which the infinite product is thus also equal (in modern terminology this is equivalent to say that the partial sum up to <math>x</math> of the harmonic series diverges asymptotically like <math>\log x</math>). Then in his second corollary Euler notes that the product

<math>\prod_{n\ge2} \frac{1}{1-\frac{1}{n^2}}</math>

converges to the finite value 2, and that there are consequently more primes than squares («  sequitur infinities plures esse numeros primos »). This proves Euclid's Theorem.[10]

Файл:Euler's infinity sign.jpg
Symbol used by Euler to denote infinity


In the same paper (Theorem 19) Euler in fact used the above equality to prove a much stronger theorem that was unknown before him, namely that the series

<math>\sum_{p\in P}\frac 1p</math>

is divergent, where Шаблон:Mvar denotes the set of all prime numbers (Euler writes that the infinite sum <math>=\log\log\infty</math>, which in modern terminology is equivalent to say that the partial sum up to <math>x</math> of this series behaves asymptotically like <math>\log\log x</math>).

Erdős's proof

Paul Erdős gave a proof[11] that also relies on the fundamental theorem of arithmetic. Every positive integer has a unique factorization into a square-free number Шаблон:Math and a square number Шаблон:Math. For example, Шаблон:Math.

Let Шаблон:Mvar be a positive integer, and let Шаблон:Mvar be the number of primes less than or equal to Шаблон:Mvar. Call those primes Шаблон:Math. Any positive integer Шаблон:Mvar which is less than or equal to Шаблон:Mvar can then be written in the form

<math>a = \left( p_1^{e_1} p_2^{e_2} \cdots p_k^{e_k} \right) s^2,</math>

where each Шаблон:Math is either Шаблон:Math or Шаблон:Math. There are Шаблон:Math ways of forming the square-free part of Шаблон:Mvar. And Шаблон:Math can be at most Шаблон:Mvar, so Шаблон:Math. Thus, at most Шаблон:Math numbers can be written in this form. In other words,

<math>N \leq 2^k \sqrt{N}.</math>

Or, rearranging, Шаблон:Mvar, the number of primes less than or equal to Шаблон:Mvar, is greater than or equal to Шаблон:Math. Since Шаблон:Mvar was arbitrary, Шаблон:Mvar can be as large as desired by choosing Шаблон:Mvar appropriately.

Furstenberg's proof

Шаблон:Main

In the 1950s, Hillel Furstenberg introduced a proof by contradiction using point-set topology.[12]

Define a topology on the integers Z, called the evenly spaced integer topology, by declaring a subset U ⊆ Z to be an open set if and only if it is either the empty set, ∅, or it is a union of arithmetic sequences S(ab) (for a ≠ 0), where

<math>S(a, b) = \{ a n + b \mid n \in \mathbb{Z} \} = a \mathbb{Z} + b. </math>

Then a contradiction follows from the property that a finite set of integers cannot be open and the property that the basis sets S(ab) are both open and closed, since

<math>\mathbb{Z} \setminus \{ -1, + 1 \} = \bigcup_{p \mathrm{\, prime}} S(p, 0)</math>

cannot be closed because its complement is finite, but is closed since it is a finite union of closed sets.

Recent proofs

Proof using the inclusion-exclusion principle

Juan Pablo Pinasco has written the following proof.[13]

Let p1, ..., pN be the smallest N primes. Then by the inclusion–exclusion principle, the number of positive integers less than or equal to x that are divisible by one of those primes is

<math>

\begin{align} 1 + \sum_{i} \left\lfloor \frac{x}{p_i} \right\rfloor - \sum_{i < j} \left\lfloor \frac{x}{p_i p_j} \right\rfloor & + \sum_{i < j < k} \left\lfloor \frac{x}{p_i p_j p_k} \right\rfloor - \cdots \\ & \cdots \pm (-1)^{N+1} \left\lfloor \frac{x}{p_1 \cdots p_N} \right\rfloor. \qquad (1) \end{align} </math>

Dividing by x and letting x → ∞ gives

<math> \sum_{i} \frac{1}{p_i} - \sum_{i < j} \frac{1}{p_i p_j} + \sum_{i < j < k} \frac{1}{p_i p_j p_k} - \cdots \pm (-1)^{N+1} \frac{1}{p_1 \cdots p_N}. \qquad (2) </math>

This can be written as

<math> 1 - \prod_{i=1}^N \left( 1 - \frac{1}{p_i} \right). \qquad (3) </math>

If no other primes than p1, ..., pN exist, then the expression in (1) is equal to <math>\lfloor x \rfloor </math> and the expression in (2) is equal to 1, but clearly the expression in (3) is not equal to 1. Therefore, there must be more primes than  p1, ..., pN.

Proof using Legendre's formula

In 2010, Junho Peter Whang published the following proof by contradiction.[14] Let k be any positive integer. Then according to Legendre's formula (sometimes attributed to de Polignac)

<math> k! = \prod_{p\text{ prime}} p^{f(p,k)} </math>

where

<math> f(p,k) = \left\lfloor \frac{k}{p} \right\rfloor + \left\lfloor \frac{k}{p^2} \right\rfloor + \cdots. </math>
<math> f(p,k) < \frac{k}{p} + \frac{k}{p^2} + \cdots = \frac{k}{p-1} \le k. </math>

But if only finitely many primes exist, then

<math> \lim_{k\to\infty} \frac{\left(\prod_p p\right)^k}{k!} = 0, </math>

(the numerator of the fraction would grow singly exponentially while by Stirling's approximation the denominator grows more quickly than singly exponentially), contradicting the fact that for each k the numerator is greater than or equal to the denominator.

Proof by construction

Filip Saidak gave the following proof by construction, which does not use reductio ad absurdum[15] or Euclid's lemma (that if a prime p divides ab then it must divide a or b).

Since each natural number greater than 1 has at least one prime factor, and two successive numbers n and (n + 1) have no factor in common, the product n(n + 1) has more different prime factors than the number n itself.  So the chain of pronic numbers:
1×2 = 2 {2},    2×3 = 6 {2, 3},    6×7 = 42 {2, 3, 7},    42×43 = 1806 {2, 3, 7, 43},    1806×1807 = 3263442 {2, 3, 7, 43, 13, 139}, · · ·
provides a sequence of unlimited growing sets of primes.

Proof using the incompressibility method

Шаблон:See Suppose there were only k primes (p1, ..., pk). By the fundamental theorem of arithmetic, any positive integer n could then be represented as <math display=block>n = {p_1}^{e_1} {p_2}^{e_2} \cdots {p_k}^{e_k},</math> where the non-negative integer exponents ei together with the finite-sized list of primes are enough to reconstruct the number. Since <math>p_i \geq 2</math> for all i, it follows that <math>e_i \leq \lg n</math> for all i (where <math>\lg</math> denotes the base-2 logarithm). This yields an encoding for n of the following size (using big O notation):

<math>O(\text{prime list size} + k \lg \lg n) = O(\lg \lg n)</math> bits.

This is a much more efficient encoding than representing n directly in binary, which takes <math>N = O(\lg n)</math> bits. An established result in lossless data compression states that one cannot generally compress N bits of information into fewer than N bits. The representation above violates this by far when n is large enough since <math>\lg \lg n = o(\lg n)</math>. Therefore, the number of primes must not be finite.[16]

Stronger results

The theorems in this section simultaneously imply Euclid's theorem and other results.

Dirichlet's theorem on arithmetic progressions

Шаблон:Main article

Dirichlet's theorem states that for any two positive coprime integers a and d, there are infinitely many primes of the form a + nd, where n is also a positive integer. In other words, there are infinitely many primes that are congruent to a modulo d.

Prime number theorem

Шаблон:Main article

Let Шаблон:Math be the prime-counting function that gives the number of primes less than or equal to Шаблон:Mvar, for any real number Шаблон:Mvar. The prime number theorem then states that Шаблон:Math is a good approximation to Шаблон:Math, in the sense that the limit of the quotient of the two functions Шаблон:Math and Шаблон:Math as Шаблон:Mvar increases without bound is 1:

<math>\lim_{x\rightarrow\infty} \frac{\pi(x)}{x/\log(x)}=1. </math>

Using asymptotic notation this result can be restated as

<math>\pi(x)\sim \frac{x}{\log x}.</math>

This yields Euclid's theorem, since <math>\lim_{x\rightarrow\infty} \frac{x}{\log x}=\infty. </math>

Bertrand–Chebyshev theorem

In number theory, Bertrand's postulate is a theorem stating that for any integer <math>n > 1</math>, there always exists at least one prime number such that

<math>n < p < 2n.</math>

Bertrand–Chebyshev theorem can also be stated as a relationship with <math>\pi(x)</math>, where <math>\pi(x)</math> is the prime-counting function (number of primes less than or equal to <math>x \,</math>):

<math>\pi(x) - \pi(\tfrac{x}{2}) \ge 1,</math> for all <math>x \ge 2.</math>


This statement was first conjectured in 1845 by Joseph Bertrand[17] (1822–1900). Bertrand himself verified his statement for all numbers in the interval Шаблон:Nowrap His conjecture was completely proved by Chebyshev (1821–1894) in 1852[18] and so the postulate is also called the Bertrand–Chebyshev theorem or Chebyshev's theorem.

Notes and references

Шаблон:Reflist

External links

Шаблон:Ancient Greek mathematics

  1. James Williamson (translator and commentator), The Elements of Euclid, With Dissertations, Clarendon Press, Oxford, 1782, page 63.
  2. Шаблон:Citation
  3. In general, for any integers a, b, c if <math>a \mid b</math> and <math>a \mid c</math>, then <math> a \mid (b - c)</math>. For more information, see Divisibility.
  4. The exact formulation of Euclid's assertion is: "The prime numbers are more numerous than any proposed multitude of prime numbers".
  5. Шаблон:Citation
  6. Michael Hardy and Catherine Woodgold, "Prime Simplicity", Mathematical Intelligencer, volume 31, number 4, fall 2009, pages 44–52.
  7. Шаблон:Citation
  8. Шаблон:Cite book
  9. Theorems 7 and their Corollaries 1 and 2 in: Leonhard Euler. Variae observationes circa series infinitas. Commentarii academiae scientiarum Petropolitanae 9, 1744, pp. 160–188. [1]. (Original) [2]. (English translation version)
  10. In his History of the Theory of Numbers (Vol. 1, p. 413) Dickson refers to this proof, as well as to another one by citing page 235 of another work by Euler: Introductio in Analysin Infinitorum. Tomus Primus. Bousquet, Lausanne 1748. [3]. There (§ 279) Euler in fact essentially restates the much stronger Theorem 19 (described below) in the paper of his former proof.
  11. Шаблон:Cite book
  12. Шаблон:Cite journal
  13. Juan Pablo Pinasco, "New Proofs of Euclid's and Euler's theorems", American Mathematical Monthly, volume 116, number 2, February, 2009, pages 172–173.
  14. Junho Peter Whang, "Another Proof of the Infinitude of the Prime Numbers", American Mathematical Monthly, volume 117, number 2, February 2010, page 181.
  15. Шаблон:Cite journal
  16. Шаблон:Citation
  17. Шаблон:Citation.
  18. Шаблон:Citation. (Proof of the postulate: 371–382). Also see Mémoires de l'Académie Impériale des Sciences de St. Pétersbourg, vol. 7, pp. 15–33, 1854